Can we use the delta-ep def of a limit to find a limiting value?

I had a random thought recently that we don't really use the delta epsilon definition for evaluating limiting values. So, I decided to have a crack at it just now. Could somebody check whether my reasoning is sound or if I've just assumed some nonsense somewhere?

**Example** Using only the definition of a limit, evaluate $$\lim_{x\to0}x=L$$ for some L. It does not matter whether the limiting value L exists or not.

$Solution.$ Let us consider the function $f:\mathbb{R}\to\mathbb{R}$ for $f(x)=x$. As per our definition for a limit, if $f$ has a $L$ as $x\to0$ then for every $\varepsilon>0$ there exists a $\delta>0$ such that for all $x\in\mathbb{R}$ $$0<|x|<\delta\ \implies\ |x-L|<\varepsilon.$$ Rewriting our first inequality using the triangle inequality give us $$|x|=|(x-L)+L|\geq||x-L|-|L||.$$ Rearranging $0<|x-L|-|L|<\delta$ gives us $$-\delta<|x-L|-|L|<0 \quad \text{or} \quad 0<|x-L|-|L|<\delta.$$ It follows that $$|L|-\delta<|x-L|<|L| \quad \text{or} \quad |L|<|x-L|<|L|+\delta$$ As such, we know $$|x-L|<|L|\ \text{or}\ |x-L|>|L|\ \implies\ |x-L|\neq|L|$$ This means either: (1) $x$ is not 0 and/or $x$ is not $2L$, or (2) $L=0$.

However, the former is a contradiction since $0<|x|<\delta$ implies $|x-L|<\varepsilon$, which would need to hold for $x=0$ since $0\in\mathbb{R}$. Therefore $L=0$ if $L$ exists. 

Final comments: My primary concern is that the definition of the function itself is not invoked when I’m manipulating the inequalities to produce some $L$. I had hoped that looking at the domain would be sufficient. So if we looked at $1/x$ instead this approach would correctly show that as $x\to0$ the function is undefined. However, the working would be identical and also redundant, since we would only be looking at the domain. Is there a legal way to do this or is this just a lost cause?

Answer

Answers can be viewed only if
  1. The questioner was satisfied and accepted the answer, or
  2. The answer was disputed, but the judge evaluated it as 100% correct.
View the answer
Erdos Erdos
4.6K
The answer is accepted.
Join Matchmaticians Affiliate Marketing Program to earn up to 50% commission on every question your affiliated users ask or answer.